Difference between revisions of "Mock AIME 6 2006-2007 Problems/Problem 10"

(Created page with "==Problem== Given a point <math>P</math> in the coordinate plane, let <math>T_k(P)</math> be the <math>90^\circ</math> rotation of <math>P</math> around the point <math>(2000-...")
 
Line 3: Line 3:
  
 
==Solution==
 
==Solution==
{{Solution}}
+
Let <math>R</math> be the rotational matrix for a point along the origin:
 +
 
 +
<math>R=\begin{pmatrix} cos(\theta) & -sin(\theta)\\ sin(\theta) & cos(\theta) \end{pmatrix}</math>
 +
 
 +
For <math>\theta = 90^\circ</math>
 +
 
 +
<math>R=\begin{pmatrix} cos(90^\circ) & -sin(90^\circ)\\ sin(90^\circ) & cos(90^\circ) \end{pmatrix}=\begin{pmatrix} 0 & -1\\ 1 & 0 \end{pmatrix}</math>
 +
 
 +
 
 +
~Tomas Diaz. orders@tomasdiaz.com

Revision as of 15:01, 25 November 2023

Problem

Given a point $P$ in the coordinate plane, let $T_k(P)$ be the $90^\circ$ rotation of $P$ around the point $(2000-k,k)$. Let $P_0$ be the point $(2007,0)$ and $P_{n+1}=T_n(P_n)$ for all integers $n\ge 0$. If $P_m$ has a $y$-coordinate of $433$, what is $m$?

Solution

Let $R$ be the rotational matrix for a point along the origin:

$R=\begin{pmatrix} cos(\theta) & -sin(\theta)\\ sin(\theta) & cos(\theta) \end{pmatrix}$

For $\theta = 90^\circ$

$R=\begin{pmatrix} cos(90^\circ) & -sin(90^\circ)\\ sin(90^\circ) & cos(90^\circ) \end{pmatrix}=\begin{pmatrix} 0 & -1\\ 1 & 0 \end{pmatrix}$


~Tomas Diaz. orders@tomasdiaz.com